You are on page 1of 184

Nature of Chapter:

1. This is an extension of Complex Numbers-1 , so you should be knowing


all concepts of that.

2. This topic is little complicated, but it becomes very easy if you understand
the basic logic of Rotation, while geometry is a std application of modulus
and argument.

3. We will relate the concepts with basic vectors to make this topic easy.
Weightage of Complex Numbers (Last 5 years)
2023 2022 2021 2020 2019 Average

JEE Main 3.8 % 3.8 % 4.1 % 4.0 % 3.3 % 3.8 %

Jee Advanced 6% 8% 6% 5% 5% 6.0 %


Complex Numbers : Geometry & Rotation

● Vector representation of complex numbers


● Geometrical representation of Modulus
● Rotation of Complex numbers
● Nth Root of Unity
Complex Numbers : Geometry & Rotation

Critical Topics:
● Geometrical representation of Modulus
Vector representation of Complex Numbers
Vector representation of complex numbers
z1 is represented by its position vector
z2 is represented by its position vector

Clearly, then will represent z2 - z1


Vector representation of complex numbers
z1 is represented by its position vector
z2 is represented by its position vector

Clearly, then will represent z2 - z1

Also,

| z1| gives length of

| z2| gives length of

| z2 - z1 | gives length of
Vector representation of complex numbers
Also,

arg(z1) gives direction of

arg(z2) gives direction of

arg(z2 - z1) gives direction of


Vector representation of complex numbers

Let us explore little bit of ‘ARGUMENT’ here


What is the Locus of z, if:
Q
(a) (b)

(c)
What is the Locus of z, if:
Q
(a)
What is the Locus of z, if:
Q
(a)

Solution:
A ray emanating from (0, 0) making angle
with the positive direction of x-axis
What is the Locus of z, if:
Q
(b)
What is the Locus of z, if:
Q
(b)

Solution:
A ray emanating from (0, 3) making angle
with the positive direction of x-axis

(0, 3)
What is the Locus of z, if:
Q
(c)
What is the Locus of z, if:
Q
(c)

Solution:
A ray emanating from (2, -5) making angle
with the positive direction of x-axis

(2, -5)
Q If , then what is the locus of z ?

A 1st quadrant

B 4th quadrant

C y=x

D Sector of a circle
Q If , then what is the locus of z ?

A 1st quadrant

B 4th quadrant

C y=x

D Sector of a circle
Q If , then what is the locus of z ?

Solution:

Since

then z is in first quadrant only


Q The shaded region is represented by

D
Q The shaded region is represented by

D
Q The shaded region is represented by

Solution:
Let z be the moving point
Then given locus

Represents rays emanating from (1, 1) and

making angle between with x-axis


Q Plot:
Q Plot:

Solution:

Included

Not included
30o 60o

(1, 1)
Geometrical representation of Modulus
Geometrical representation of Modulus

We know
|z1| → distance of z1 from origin.
|z2 - z1| → distance between z1 & z2
Let z1 = x1 + iy1 & z2 = x2 + iy2 to observe above statements
Find the Locus of z, if:
Q
(a) |z| = 2 (b) |z + 2 - 3i| = 1
(c) |z - 4i| = |z + 3 + 4i| (d) |z - (1 + 2i)| = |z - (-1)|
Find the Locus of z, if:
Q
(a) |z| = 2
Find the Locus of z, if:
Q
(a) |z| = 2

Solution:

Method I
Given, |z| = 2
Let z = x + iy

⸫ Locus of z is circle with centre (0, 0)


and radius 2
Find the Locus of z, if:
Q
(a) |z| = 2

Solution:

Method II:
Given, |z - 0| = 2
It represent distance of z from origin is constant
and is equal to 2 which is locus of circle with

centre (0, 0) and radius 2


Find the Locus of z, if:
Q
(b) |z + 2 - 3i| = 1
Find the Locus of z, if:
Q
(b) |z + 2 - 3i| = 1

Solution:
Method I:
Given, |z + 2 - 3i| = 1
Let z = x + iy

⸫ Locus represents circle with centre (-2, 3)


and radius 1
Find the Locus of z, if:
Q
(b) |z + 2 - 3i| = 1

Solution:
Method II:
Given, |z + 2 - 3i| = 1
⇒ |z - (-2 + 3i)| = 1

It represents distance of z from (-2 + 3i) is


constant and is equal to 1 which is locus of
circle with centre (-2, 3) and radius 1
Find the Locus of z, if:
Q
(c) |z - 4i| = |z + 3 + 4i|
Find the Locus of z, if:
Q
(c) |z - 4i| = |z + 3 + 4i|

Solution:
Method I
Find the Locus of z, if:
Q
(c) |z - 4i| = |z + 3 + 4i|

Solution:

Which is perpendicular bisector of line joining


(0, 4) and (-3, -4)
Find the Locus of z, if:
Q
(c) |z - 4i| = |z + 3 + 4i|

Solution:
Method II
Given, |z - 4i| = |z + 3 + 4i|
As z is equidistant from A (0, 4) and B (-3, -4)
so locus of z is perpendicular bisector of line
segment joining A and B
Find the Locus of z, if:
Q
(d) |z - (1 + 2i)| = |z - (-1)|
Find the Locus of z, if:
Q
(d) |z - (1 + 2i)| = |z - (-1)|

Solution:
Method I
Given, |z -(1 + 2i)| = |z -(-1)|
Let z = x + iy

Squaring on both sides, we get


Find the Locus of z, if:
Q
(d) |z - (1 + 2i)| = |z - (-1)|

Solution:

which is straight line which is perpendicular


bisector of line joining (1, 2) and (−1, 0)
Find the Locus of z, if:
Q
(d) |z - (1 + 2i)| = |z - (-1)|

Solution:
Method II
Given, |z - (1 + 2i)| = |z -(-1)|
As z is equidistant from A (1, 2) and B (-1, 0)
so locus of z is perpendicular bisector of line
segment joining A and B
JEE Main 24th June, 2022

Q Let A = {z ∈ C : 1 ≤ |z - (1 + i)| ≤ 2}
and B = {z ∈ A : |z - (1 - i)| = 1}. Then B :

A is an empty set

contains exactly two


B elements

C contains exactly three


elements

D is an infinite set
JEE Main 24th June, 2022

Q Let A = {z ∈ C : 1 ≤ |z - (1 + i)| ≤ 2}
and B = {z ∈ A : |z - (1 - i)| = 1}. Then B :

A is an empty set

contains exactly two


B elements

C contains exactly three


elements

D is an infinite set
Solution:

It represent two concentric circles both having center at (1, 1)


and radius 1 and 2. Also given,

It represent a circle having center at (1, -1) and radius 1.


In the common region, infinite values of B are possible.
JEE Main 26th July, 2022

If z = x + iy satisfies |z| - 2 = 0 and |z - i| - | z + 5i| = 0,


Q
then

A x + 2y - 4 = 0

B x2 + y - 4 = 0

C x + 2y + 4 = 0

D x2 - y + 3 = 0
JEE Main 26th July, 2022

If z = x + iy satisfies |z| - 2 = 0 and |z - i| - | z + 5i| = 0,


Q
then

A x + 2y - 4 = 0

B x2 + y - 4 = 0

C x + 2y + 4 = 0

D x2 - y + 3 = 0
JEE Main 26th July, 2022

If z = x + iy satisfies |z| - 2 = 0 and |z - i| - | z + 5i| = 0,


Q
then

Solution:
Q

A 2

B 2i

C 1

D -i
Q

A 2

B 2i

C 1

D -i
Q

Solution:
Given, |z - i | = 1, which represents a circle with
centre (0, 1) and radius 1 .

Also, given , which represents positive


y-axis.

Thus, point A and B are their intersection points


But argument of A is not defined
Thus only one point B (0, 2)
∴ z = 2i
Q If |z + 3 - i| = 4 & , then z is

A 1+i

B 1 + i or 3 - 3i

C - 1 + i or 3 + 3i

D - 3 - 3i
Q If |z + 3 - i| = 4 & , then z is

A 1+i

B 1 + i or 3 - 3i

C - 1 + i or 3 + 3i

D - 3 - 3i
Solution:
Given, |z + 3 - i| = 4
which is circle with center at (−3, 1) and radius 4
Also given , which is a ray emanating from origin

and making angle with positive direction of x-axis

Thus A is the point of intersection which lies in first quadrant


So equation of circle is
(x + 3)2 + (y - 1)2 = 42
x2 + 9 + 6x + y2 + 1 - 2y = 16

Solving with y = x, we get


x2 + 6x + x2 - 2x - 6 = 0
Solution:

⇒ 2x2 + 4x - 6 = 0
⇒ x2 + 2x - 3 = 0
⇒ (x + 3) (x - 1) = 0
∴ x = -3, x = 1
But A ∈ 1st quadrant
Thus, x = 1 and y = x = 1
⸫ Point is z = 1 + i
Find complex number having least positive argument
Q
and satisfying |z - 5i| ≤ 3
Find complex number having least positive argument
Q
and satisfying |z - 5i| ≤ 3

Solution:

Given, |z - 5i| ≤ 3
which represents circle with centre (0, 5) and radius 3
Observe that OA will have least positive argument
For calculating A
Here in ΔOAC,
OA2 + AC2 = OC2
OA2 = 25 - 9
OA = 4
Now, and
Find complex number having least positive argument
Q
and satisfying |z - 5i| ≤ 3

Solution:
Now, in ΔAOP,
Q Shaded region is given by

D None of these
Q Shaded region is given by

D None of these
Solution:
Let A(z1) & B(z2) be fixed points and P(z) moves in plane such that

|z - z1| + |z - z2| = k

k > |z - z2| k = |z - z2| k < |z - z2|


Ellipse line AB No locus
Slide 63

1 discuss:
yahan Locus of P likhna hai when PS + PS' = k const hota hai, for various values of k
Arvind Kalia, 02-10-2023
Find Locus of z, if:
Q
(a) |z - 1| + |z + 1| = 2 (b)

(c) |z - 4i| + |z + 4i| = 10


Find Locus of z, if:
Q
(a) |z - 1| + |z + 1| = 2
Find Locus of z, if:
Q
(a) |z - 1| + |z + 1| = 2

Solution:

Give, |z - 1| + |z + 1| = 2
Here, sum of distance of z from (1, 0) and (-1, 0)
is equal to distance between (1, 0) and (-1, 0)
Thus, the locus is a line segment joining (1, 0)
and (-1, 0)
Find Locus of z, if:
Q
(b)
Find Locus of z, if:
Q
(b)

Solution:
Given,

Here sum of distance of z from (1, 0) and (-1, 0)


is and distance between (1, 0) and (-1, 0) is 2

Thus, there is no locus, as sum of two sides of


triangle cannot be less than third side
Find Locus of z, if:
Q
(c) |z - 4i| + |z + 4i| = 10
Find Locus of z, if:
Q
(c) |z - 4i| + |z + 4i| = 10

Solution:

Given, |z - 4i| + |z + 4i| = 10


i.e., |z - 4i| + |z - (-4i)| = 10

Here sum of distance of z from (0, 4) and (0, -4)


is 10 and distance between (0, 4) and (0, -4) is 8

Thus, locus of z is ellipse


If ‘z’ be any complex number such that |3z - 2| + |3z + 2| = 4,
Q
then identify the locus of ‘z’.
If ‘z’ be any complex number such that |3z - 2| + |3z + 2| = 4,
Q
then identify the locus of ‘z’.

Solution:
Let A(z1) & B(z2) be fixed points and P(z) moves in plane such that

|z - z2| - |z - z1| = k

k < |z1 - z2|


k > |z1 - z2|
Hyperbola
k = |z - z2| No locus
line Joining AB but
excluding segment AB
Slide 73

2 discuss:
yahan Locus of P likhna hai when PS - PS' = k const hota hai, for various values of k
Arvind Kalia, 02-10-2023
Find Locus of z, if:
Q
(a) |z - 3 + 4i| - |z + 2 - i| = 1

(b) |z - 1| - |z + 1| = 4
Find Locus of z, if:
Q
(a) |z - 3 + 4i| - |z + 2 - i| = 1
Find Locus of z, if:
Q
(a) |z - 3 + 4i| - |z + 2 - i| = 1

Solution:

|z - 3 + 4i| - |z + 2 - i| = 1
⇒ |z - (3 - 4i)| - |z - (-2 + i)| = 1

Here, difference of distance of z from A(3, -4) and


B(-2, 1) is 1 and distance between A(3, -4) and
B(-2, 1) is 5√2
Thus, |PA - PB| < AB
Hence, locus of z is hyperbola
Find Locus of z, if:
Q
(b) |z - 1| - |z + 1| = 4
Find Locus of z, if:
Q
(b) |z - 1| - |z + 1| = 4

Solution:
Given, |z - 1| - |z + 1| = 4
Here, difference of distance of z from (1, 0) and (-1, 0)
is 4 and distance between (1, 0) and (-1, 0) is 2
Thus, there is no locus, as difference of two sides of
triangle cannot be bigger than third side
JEE Main 25th Jan, 2023

hyperbola with the length of the


A transverse axis 7

B Hyperbola with eccentricity 2

straight line with the sum of its intercepts


C on the coordinate axes equals - 18
straight line with the sum of its intercepts
D
on the coordinate axes equals 14
JEE Main 25th Jan, 2023

hyperbola with the length of the


A transverse axis 7

B Hyperbola with eccentricity 2

straight line with the sum of its intercepts


C on the coordinate axes equals - 18
straight line with the sum of its intercepts
D
on the coordinate axes equals 14
JEE Main 25th Jan, 2023

Solution:
Remark

Locus of P such that PA/PB = k, where A and B are given points is:
1) perpendicular bisector of AB if k = 1
2) a circle if k is not equal to 1
Slide 82

3 Remark add krna hai:


Locus of P such that PA/PB = k, where A and B are given points is:
1) perp bisector of AB if k=1
2) a circle if k is not equal to 1
Arvind Kalia, 02-10-2023
Q If z = x + iy then the equation does not
represent a circle when

B m=1

C m=2

D m=3
Q If z = x + iy then the equation does not
represent a circle when

B m=1

C m=2

D m=3
Q If z = x + iy then the equation does not
represent a circle when

Solution:
Q Find locus of z if |z| = 2 |z - 1|.
Q Find locus of z if |z| = 2 |z - 1|.

Solution:
Q If z is a complex number, then locus of z satisfying
the condition |2z - 1| = |z - 1| is

Perpendicular bisector of
A line joining (½, 0) and (1, 0)

B circle

C parabola

D None of these
Q If z is a complex number, then locus of z satisfying
the condition |2z - 1| = |z - 1| is

Perpendicular bisector of
A line joining (½, 0) and (1, 0)

B circle

C parabola

D None of these
Q If z is a complex number, then locus of z satisfying
the condition |2z - 1| = |z - 1| is

Solution:
JEE Main 25th Jan, 2023

Q Let ‘ z’ be a complex number such that

Then z lies on the circle of radius 2 and centre :

A (0, -2)

B (0, 0)

C (0, 2)

D (2, 0)
JEE Main 25th Jan, 2023

Q Let ‘ z’ be a complex number such that

Then z lies on the circle of radius 2 and centre :

A (0, -2)

B (0, 0)

C (0, 2)

D (2, 0)
Solution:
JEE Main 1st Feb, 2023

Q If the center and radius of the circle are

respectively (𝞪, β) and γ, then 3(𝞪 + β+ γ) is equal to

A 12

B 10

C 11

D 9
JEE Main 1st Feb, 2023

Q If the center and radius of the circle are

respectively (𝞪, β) and γ, then 3(𝞪 + β+ γ) is equal to

A 12

B 10

C 11

D 9
Solution:
Solution:
JEE Main 6th Apr, 2023

Q For 𝞪,β, z ∈ С and 𝞴 > 1, if is the radius of the circle


|z - 𝞪|2 + |z - β|2 = 2𝞴, then | 𝞪 - β| is equal to ____.
JEE Main 6th Apr, 2023

Q For 𝞪,β, z ∈ С and 𝞴 > 1, if is the radius of the circle


|z - 𝞪|2 + |z - β|2 = 2𝞴, then | 𝞪 - β| is equal to ____.

Ans: 2
Solution:
Let’s do some more examples based upon geometrical interpretations
Q Find maximum & minimum value of
(a) |z - i| if | z - 4i | = 2
(b) |z + 4| if |z - (-4 + 3i)| = 2
Solution:

(a) (b)
Q Find maximum & minimum value of |z - (3 + 4i)|
if |z - 1| = 2
Q Find maximum & minimum value of |z - (3 + 4i)|
if |z - 1| = 2

Solution:
Given, |z -1| = 2
which is a circle with centre C(1, 0) and radius r = 2
Thus z will lie on circle and |z - (3 + 4i)| is the distance of
point on circle from P(3, 4)
⸫ Maximum distance

Minimum distance
JEE Main 2014

Q If z is a complex number such that |z| ≥ 2, then the

minimum value of

A is strictly greater than

B is strictly greater than but less than

C is equal to

D Lies in the interval (1, 2)


JEE Main 2014

Q If z is a complex number such that |z| ≥ 2, then the

minimum value of

A is strictly greater than

B is strictly greater than but less than

C is equal to

D Lies in the interval (1, 2)


Solution:
Rotation of Complex numbers
Rotation of complex numbers

Rotation of Complex Number (Coni’s Method)


Rotation of complex numbers

Observation

1. If z2 = z1eiθ then
Rotation of complex numbers

Observation

1. If z2 = z1eiθ then
JEE Main 11th April, 2023

Q Let w1 be the point obtained by the rotation of z1 = 5 + 4i about


the origin through a right angle in the anticlockwise direction,
and w2 be the point obtained by the rotation of z2 = 3 + 5i about
the origin through a right angle in the clockwise direction. Then
the principal argument of w1 — w2 is equal to:

D
JEE Main 11th April, 2023

Q Let w1 be the point obtained by the rotation of z1 = 5 + 4i about


the origin through a right angle in the anticlockwise direction,
and w2 be the point obtained by the rotation of z2 = 3 + 5i about
the origin through a right angle in the clockwise direction. Then
the principal argument of w1 — w2 is equal to:

D
Solution:

Given z1 = 5 + 4i.
When you rotate z1 by 90° anticlockwise about the origin, the real part becomes
negative of the imaginary part of z1 and the imaginary part becomes the real part of z1.
Therefore, w1 = – 4 + 5i
Z2 = 3 + 5i
When you rotate z2 by 90° clockwise about the origin, the real part becomes the
imaginary part of z2, and the imaginary part becomes negative of the real part of z2.
Therefore, w2 = 5 - 3i
w1 - w2 = (- 4 + 5i) - (5 - 3i)
w1 - w2 = -9 + 8i
Argument =
Q If ΔAOB is an isosceles right angled triangle then
prove that z12 + z22 = 0 , where O is origin.
Q If ΔAOB is an isosceles right angled triangle then
prove that z12 + z22 = 0 , where O is origin.

Solution:
Here,

Squaring on both sides


Q The centre of a regular polygon of n sides is located at
the point z = 0, and one of its vertices z1 is known. If z2
be the vertex adjacent to z1, then find z2 in terms of z1 .
Solution:
B(z2) O

A(z1) B(z2)
Q Prove that if the complex numbers z1, z2 and the origin
form an equilateral triangle, then
Q Prove that if the complex numbers z1, z2 and the origin
form an equilateral triangle, then

Solution:
l
z1 z2

l l
60
o

Origin
Q Prove that if the complex numbers z1, z2 and the origin
form an equilateral triangle, then

Solution:
Rotation of complex numbers

Observation

2. If z3 - z1 = (z2 - z1)eiθ , then


Rotation of complex numbers

Observation

2. If z3 - z1 = (z2 - z1)eiθ , then

i.e. z3 − z1 is obtained by rotating z2 − z1


anticlockwise by ∠θ
Q If z1, z2, z3 are vertices of an isosceles triangle, right
angled at z2 then prove that z12 + 2z22 + z32 = 2z2 (z1 + z3)
Q If z1, z2, z3 are vertices of an isosceles triangle, right
angled at z2 then prove that z12 + 2z22 + z32 = 2z2 (z1 + z3)

Solution:
Here,

Squaring both sides, we get


If one vertex of the triangle having maximum area
Q
that can be inscribed in the circle |z - i| = 5 is 3 - 3i,
then find the other vertices of the triangle.
If one vertex of the triangle having maximum area
Q
that can be inscribed in the circle |z - i| = 5 is 3 - 3i,
then find the other vertices of the triangle.

Solution:
If z1, z2 and z3 are the vertices of ΔABC, which is not right angled
Q
triangle taken in anticlockwise direction and z0 is the circumcentre

then is equal to

A 0

B 1

C -1

D 2
If z1, z2 and z3 are the vertices of ΔABC, which is not right angled
Q
triangle taken in anticlockwise direction and z0 is the circumcentre

then is equal to

A 0

B 1

C -1

D 2
Solution:
A(z1)

O(z0)

B(z2)
C(z3)
Result

z1, z2 & z3 are vertices of an equilateral triangle if

1.

2.
Q If 1 + i & 3 + 3i are two vertices of an equilateral
triangle then find third possible vertex.
Q If 1 + i & 3 + 3i are two vertices of an equilateral
triangle then find third possible vertex.

Solution:
Here,
Q If z, - z and 1 - z are the vertices of an equilateral
triangle, then Re(z) =

A 1

D
Q If z, - z and 1 - z are the vertices of an equilateral
triangle, then Re(z) =

A 1

D
Q If z, - z and 1 - z are the vertices of an equilateral
triangle, then Re(z) =

Solution:
Q Let z1 and z2 be the complex roots of the equation 3z2 + 3z + b = 0.
If the origin, together with the points represented by z1 and z2
from an equilateral triangle, then the value of b is

A -1

B 0

C 1

D Can not be decided


Q Let z1 and z2 be the complex roots of the equation 3z2 + 3z + b = 0.
If the origin, together with the points represented by z1 and z2
from an equilateral triangle, then the value of b is

A -1

B 0

C 1

D Can not be decided


Q Let z1 and z2 be the complex roots of the equation 3z2 + 3z + b = 0.
If the origin, together with the points represented by z1 and z2
from an equilateral triangle, then the value of b is

Solution:
Q Consider a square ABCD such that Z1, Z2, Z3 and Z4
represent its vertices A, B, C and D respectively.
Express ‘Z3’ and ‘Z4’ in terms of ‘Z1’ and ‘Z2’.
Q Consider a square ABCD such that Z1, Z2, Z3 and Z4
represent its vertices A, B, C and D respectively.
Express ‘Z3’ and ‘Z4’ in terms of ‘Z1’ and ‘Z2’.

Solution:
Consider the rotation of AB about A through an
angle π/4, we get

⇒ Z3 = Z1 + ( Z2 — Z1)( 1 + i)

Similarly,

⇒ Z4 = Z1 + i (Z2 — Z1)
Rotation of complex numbers

Observation

3. If , then
Rotation of complex numbers

Observation

3. If , then

we have to move from z2 − z1 in anticlockwise direction by


∠θ to reach z3 − z2.

In this case nothing can be concluded by their lengths.


Rotation of complex numbers

Result

If then locus of z is

an arc of the circle, major or minor


depending upon θ is acute of obtuse.
Q The locus of the points z which satisfy the condition

A a straight line

B a circle

C a parabola

D None of these
Q The locus of the points z which satisfy the condition

A a straight line

B a circle

C a parabola

D None of these
Solution:
JEE Main 26th Aug, 2021

Q The equation, arg represents a circle with :

A Centre at (0, - 1) and radius √2

B Centre at (0, 1) and radius √2

C Centre at (0, 0) and radius √2

D Centre at (0, 1) and radius 2


JEE Main 26th Aug, 2021

Q The equation, arg represents a circle with :

A Centre at (0, - 1) and radius √2

B Centre at (0, 1) and radius √2

C Centre at (0, 0) and radius √2

D Centre at (0, 1) and radius 2


Solution:
Q A necessary and sufficient condition for the points z1, z2 and z3

to be collinear is that the complex number

A real

B imaginary

C purely imaginary

D of the form λ(1 + i), λ ∈ R - {0}


Q A necessary and sufficient condition for the points z1, z2 and z3

to be collinear is that the complex number

A real

B imaginary

C purely imaginary

D of the form λ(1 + i), λ ∈ R - {0}


Q A necessary and sufficient condition for the points z1, z2 and z3

to be collinear is that the complex number

Solution:
Observation

If z1, z2, z3, z4 in order are vertices of square then:


Observation

If z1, z2, z3, z4 in order are vertices of square then:


Nth Roots of Unity
Nth roots of unity

zn = 1
i.e. zn = e2πki

Clearly, these roots when plotted divides unit


circle in ‘n’ equal parts.
Nth roots of unity
Properties
If 1, ⍺1, ⍺2, ….., ⍺n - 1 are nth roots of unity i.e. roots of zn = 1, then:

(a) 1 + ⍺1 + ⍺2 + … + ⍺n - 1 = 0

(b) 1, ⍺1, ⍺2, ..…, ⍺n - 1 form a GP

(c) zn - 1 = (z - 1)(z - ⍺1)(z - ⍺2) × …….. × (z - ⍺n - 1)


Nth roots of unity

Observation

Taking limit as z ⟶ 1 on both sides:


Q If 𝞪 is the fifth root of unity then

D
Q If 𝞪 is the fifth root of unity then

D
Q If 𝞪 is the fifth root of unity then

Solution:
JEE Main 25th July, 2022

Q If ⍺, β, γ, δ are the roots of the equation x4 + x3 + x2 + x + 1 = 0,


then ⍺2021 + β2021 + γ2021 + δ2021 is equal to

A -4

B -1

C 1

D 4
JEE Main 25th July, 2022

Q If ⍺, β, γ, δ are the roots of the equation x4 + x3 + x2 + x + 1 = 0,


then ⍺2021 + β2021 + γ2021 + δ2021 is equal to

A -4

B -1

C 1

D 4
JEE Main 25th July, 2022

Q If ⍺, β, γ, δ are the roots of the equation x4 + x3 + x2 + x + 1 = 0,


then ⍺2021 + β2021 + γ2021 + δ2021 is equal to

Solution:

⍺, β, γ, δ root of the equation x4 + x3 + x2 + x + 1 = 0


Which are 5th roots of unity except 1.
Then ⍺2021 + β2021 + γ2021 + δ2021 = ⍺ + β + γ + δ = -1
Q Find roots of the following: z5 + z4 + z3 + z2 + z + 1 = 0
Q Find roots of the following: z5 + z4 + z3 + z2 + z + 1 = 0

Solution:
Given,
Q Find roots of the following: z5 + z4 + z3 + z2 + z + 1 = 0

Solution:
Putting k = 0, 1, 2, 3, 4, 5

But z ≠ 1
Evaluate:
Q
(a)

(b)

(c)
Evaluate:
Q
(a)
Solution:

Given,

We know,
Evaluate:
Q
(b)
Solution:

Given,
IIT 1998
Evaluate:
Q
(c)
Solution:

Given,

Multiply and divide by ‘i’


JEE Advanced 2015

Q For any integer k, let where

The value of the expression


JEE Advanced 2015

Q For any integer k, let where

The value of the expression

Ans : 4
Solution:

You might also like